Эквивалентность одного квантового потока и нулевого потока

В обзоре квантового эффекта Холла Эди Стерн говорит о системе квантового зала: «Спектр в Φ "=" Φ 0 совпадает со спектром в Φ "=" 0 ..." Может ли кто-нибудь объяснить, почему это так? Похоже, что приложенное магнитное поле определенно изменяет гамильтониан и, следовательно, спектр, но, по-видимому, не тогда, когда поток представляет собой один квант.

Также я приношу свои извинения за любые ошибки новичка или, если на это есть ответ в другом месте, я новичок в stackexchange. Спасибо.

Какой обзор? Этот ?
Да, это тот

Ответы (2)

Государства в Φ и Φ + Φ 0 связаны калибровочным преобразованием [1], поэтому спектр должен быть одинаковым. Для конкретности поговорим об электронах, закрепленных на кольце радиусом р . Параметризируйте волновую функцию ψ по длине дуги л . Периодичность требует, чтобы

ψ ( л + 2 π р ) "=" ψ ( л ) .
Есть какой-то гамильтониан ЧАС на ринге - ЧАС должен быть калибровочно-инвариантным, поэтому он является функцией только положения л и калибровочно-инвариантная производная Д "=" я л + А , где А это датчик.

Теперь займемся калибровочным преобразованием А А + н / р , ψ е я н л / р ψ . Это касается периодической границы тогда и только тогда, когда н является целым числом, поэтому он переводит собственные состояния гамильтониана в собственные состояния гамильтониана. Теперь, используя соотношение

А д л "=" С × А д а "=" С Б д а Φ ,
мы видим, что наше калибровочное преобразование эквивалентно изменению кванта потока на Φ 0 .

Обратите внимание, что мы могли бы также попробовать калибровочное преобразование вида А А + н + θ / 2 π р . Мы можем делать это до тех пор, пока меняем наше условие связи на

ψ ( л + 2 π р ) "=" е я θ ψ ( л ) .
В частности, мы можем преобразовать калибровочное поле в нуль и остаться без А но с нетривиальным граничным условием выше. Так что, если хотите, вы можете забыть о магнитном поле и калибровочном поле и думать вместо этого о незаряженной задаче, но с «скрученным» граничным условием. Это должно иметь смысл - в нем нет магнетизма. д "=" 1 так что я должен быть в состоянии в основном избавиться от А . Краевое условие — единственный остаток.

[1] Я называю это калибровочным преобразованием, но в физическом случае кольца, пронизанного потоком, это калибровочное преобразование нельзя распространить на все пространство. Это следует из того факта, что он изменяет количество потока, пронизывающего кольцо, но это калибровочно инвариантно. Однако электронный спектр не знает, что это преобразование нельзя распространить на все пространство, и поэтому оно все еще работает.

Это происходит из-за адиабатического гипотезиса: если процедура введения кванта потока в систему адиабатическая и система находится в собственном состоянии гамильтониана ψ н с собственной энергией Е н затем он останется заблокированным в этом собственном состоянии (вплоть до глобального фазового коэффициента) во время и в конце процесса.

Утверждение касается спектра, а не состояния